Đến nội dung

cvp nội dung

Có 411 mục bởi cvp (Tìm giới hạn từ 29-04-2020)



Sắp theo                Sắp xếp  

#293549 Topic bất đẳng thức THCS (2)

Đã gửi bởi cvp on 12-01-2012 - 20:31 trong Bất đẳng thức và cực trị

Các anh em nhân tiện giúp em bài này nha :icon6:
Bài 54:
Cho các số thực không âm $x,y,z$ thỏa mãn điều kiện $x+y+z=1$. Chứng minh rằng:
$\frac{x^{2}+1}{y^{2}+1}+\frac{y^{2}+1}{z^{2}+1}+\frac{z^{2}+1}{x^{2}+1}\leq \frac{7}{2}$ :B):



#293552 Topic bất đẳng thức THCS (2)

Đã gửi bởi cvp on 12-01-2012 - 20:40 trong Bất đẳng thức và cực trị

không biết ý kiến 2 anh ấy thế nào? :icon14: còn em thì đồng ý cả 2 tay ^_^
...............................................................................................................
VÌ TOPIC "BẤT ĐẲNG THỨC THCS (2)". :lol:



#293489 Topic bất đẳng thức THCS (2)

Đã gửi bởi cvp on 12-01-2012 - 15:48 trong Bất đẳng thức và cực trị

Tặng topic anh Kiên một bài! :icon6:
Bài 53: Chứng minh bất đẳng thức sau với mọi x,y>0:
$\frac{2x^{2}+3y^{2}}{2x^{3}+3y^{3}}+\frac{2y^{2}+3x^{2}}{2y^{3}+3x^{3}}\leq \frac{4}{x+y}$
Dấu "=" xảy ra khi nào?



#318846 Topic bất đẳng thức THCS (2)

Đã gửi bởi cvp on 23-05-2012 - 20:43 trong Bất đẳng thức và cực trị

Tặng topic này 1 bài :D!
Bài 366:
CMR: $\sum_{k=1}^{n}\sqrt{(1+k)^k}<(n+1)! (1)$


Bài giải:
$n=1 \sqrt{2} < 2!=2$. Suy ra $(1)$ đúng với $n=1$.
Giả sử $(1)$ đúng với $n$, ta phải chứng minh $(1)$ đúng với $n+1$.
Ta có:
$\sum_{k=1}^{n+1}\sqrt{(1+k)^k}=\sum_{k=1}^{n}\sqrt{(1+k)^k}+\sqrt{(n+2)^{n+1}}<(n+1)!+\sqrt{(n+2)^{n+1}}.$
Ta cần CM:
$(n+1)!+\sqrt{(n+2)^{n+1}}< (n+2)! \Leftrightarrow \sqrt{(n+2)^{n+1}}<(n+1)(n+1)!$
Mặt khác: $\sqrt{n^n}<n! \forall \in \mathbb{N}$ ( VMF ta pro chứng minh cái này dễ :P).
Nên: $\sqrt{(n+2)^{n+1}}< \frac{(n+2)!}{\sqrt{n+2}}=(n+1)!\sqrt{n+2}<(n+1)!(n+1).$
Vậy $(1)$ đúng $\forall n \in \mathbb{N}$.



#291735 Topic bất đẳng thức THCS (2)

Đã gửi bởi cvp on 02-01-2012 - 21:00 trong Bất đẳng thức và cực trị

link bài 13 này mọi người! zo chơi thử không bjt aj hỏi kệ cứ post cho ae đọc! :icon6:
xem ở đây:http://www.artofprob...?f=151&t=455771



#320119 Topic tỉ lệ thức THCS

Đã gửi bởi cvp on 27-05-2012 - 21:55 trong Đại số

topic vắng vẻ quá xin đóng góp 1 bài vậy:
Cho biểu thức: $P=\frac{x+y}{z+t}+\frac{y+z}{t+x}+\frac{z+t}{x+y}+\frac{x+t}{z+y}$
Tìm giá trị của P biết rằng:
$\frac{x}{y+z+t}=\frac{y}{z+t+x}=\frac{z}{t+x+y}=\frac{t}{x+y+z}$


Áp dụng tính chất tỉ lệ thức ta có: $\large \frac{x}{y+z+t}=\frac{y}{z+t+x}=\frac{z}{t+x+y}=\frac{t}{x+y+z}=\frac{x+y+z+t}{3(x+y+z+t)}=\frac{1}{3}$
Suy ra $\large \begin{cases} &3x=y+z+t(1)\\ &3y=x+z+t(2)\\ &3z=x+y+t(3)\\ &3t=x+y+z(4) \end{cases}$.
Từ $(1);(2) \Rightarrow x+y=z+t (*1)$.
Mặt khác từ $\large (1);(4)\Rightarrow x+t=y+z (*2)$
Từ $\large (*1); (*2)\Rightarrow x=z$. Tương tự ta có được $x=y=z=t \Rightarrow P=4$.



#293727 Topic các bất đẳng thức lớp 8 hay dùng và các bài toán BĐT

Đã gửi bởi cvp on 13-01-2012 - 19:29 trong Bất đẳng thức và cực trị

Tặng anh em topic này mọt bài dễ đây :icon6:
Bài 2: (trả rõ là bài mấy nữa gọi tạm là bài 2 vậy :P)
Cho các số $a,b,c,d \in \mathbb{Z} $. Chứng minh rằng:
$\frac{a^{2}}{b^{5}}+\frac{b^{2}}{c^{5}}+\frac{c^{2}}{d^{5}}+\frac{d^{2}}{a^{5}}\geq \frac{1}{a^{3}}+\frac{1}{b^{3}}+\frac{1}{c^{3}}+\frac{1}{d^{3}}$
P/s: Do sự cố kĩ thuật (bài này khá dễ) nên các cao thủ có lv > THCS đừng chém để các bạn THCS làm nha :)



#293878 Topic các bất đẳng thức lớp 8 hay dùng và các bài toán BĐT

Đã gửi bởi cvp on 14-01-2012 - 19:25 trong Bất đẳng thức và cực trị

Bài 3:
Với $a,b,c>1$, chứng minh rằng:
$P=(\frac{2+b+c}{1+a})^{2}+(\frac{2+c+a}{1+b})^{2}+(\frac{2+a+b}{1+c})^{2}\geq 12$
:icon6:



#293926 Topic các bất đẳng thức lớp 8 hay dùng và các bài toán BĐT

Đã gửi bởi cvp on 14-01-2012 - 23:36 trong Bất đẳng thức và cực trị

Bài 3:
đây là cách của em.
Ta có bài toán phụ : $\frac{2+b+c}{1+a}+\frac{2+c+a}{1+b}+\frac{2+a+b}{1+c}\geq 6$
CM:
$\frac{2+b+c}{1+a}+1+\frac{2+c+a}{1+b}+1+\frac{2+a+b}{1+c}+1\geq 9$
$\Leftrightarrow (3+a+b+c)(\frac{1}{1+a}+\frac{1}{1+b}+\frac{1}{1+c})\geq 9$
Ta có:
$3+a+b+c\geq 3(1+\sqrt[3]{abc}) (1)$
$\frac{1}{1+a}+\frac{1}{1+b}+\frac{1}{1+c}\geq \frac{3}{1+\sqrt[3]{abc}}$ (2)
nhân vế vs vế ta được bđt phụ.

Quay lại bài toán ta có:
$P\geq 3.(\frac{\frac{2+b+c}{1+a}+\frac{2+c+a}{1+b}+\frac{2+a+b}{1+c}}{3})^{2}\geq 3.(\frac{6}{3})^{2}=12$
Ta được ĐPCM. ($\square$)



#320804 TỤ HỌP CỦA MA CŨ VÀ MA MỚI VÀO : D

Đã gửi bởi cvp on 30-05-2012 - 10:46 trong Góc giao lưu

hì, nick này là của anh em cho ( đỡ phải tạo :P)!
Diễn đàn nhiều VP nhưng có vẻ VP rất ít onl có mỗi em rảnh hay sao ý :(.



#314030 TỤ HỌP CỦA MA CŨ VÀ MA MỚI VÀO : D

Đã gửi bởi cvp on 02-05-2012 - 22:54 trong Góc giao lưu

Tên: Đinh Công Quý
Sinh ngày: 02-11-1998
Y!M: [email protected]
Nơi ở: Tam Dương-Vĩnh Phúc.
Trường: THCS Tam Dương.
Sở thích: xem Pokemon + sưu tầm + nghe nhạc.
Sở đoảng: Em chịu :|.
Ai là dân Vĩnh Phúc ép nick Y!M của em đê, tìm mãi được có 2 em :(, dân VP hiếm quá.



#201843 Gõ công thức toán học lên diễn đàn bằng Mathtype

Đã gửi bởi cvp on 19-06-2009 - 07:58 trong Công thức Toán trên diễn đàn

$\dfrac{{ - b \pm \sqrt {b^2 - 4ac} }}{{2a}}$



#203277 Bất đẳng thức

Đã gửi bởi cvp on 28-06-2009 - 21:47 trong Bất đẳng thức - Cực trị

bài nè hình như là bài NMO của Việt Nam năm 2008 :pe



#203254 Bất đẳng thức

Đã gửi bởi cvp on 28-06-2009 - 20:19 trong Bất đẳng thức - Cực trị

Cho $ a,b,c $ đôi một khác nhau: CMR:$ (a^{2}+b^{2}+c^{2})(\dfrac{1}{(a-b)^{2}}+\dfrac{1}{(b-c)^{2}}+\dfrac{1}{(c-a)^{2}})\geq \dfrac{9}{2}.$

Tui ddaay góp vui cách khác:
Sử dụng bđt Đào Hải Long;
$\dfrac{a^2}{(b-c)^2}+\dfrac{b^2}{(c-a)^2}+\dfrac{c^2}{(a-b)^2}\ge2$ (1)
cm đơn giản bằng cách đặt $x=\dfrac{a}{b-c};y=\dfrac{b}{c-a};z=\dfrac{c}{a-b}$
Tiếp theo chứng minh theo cách tương tự:
$\dfrac{(a+b)^2}{(a-b)^2}+\dfrac{(b+c)^2}{(b-c)^2}+\dfrac{(c+a)^2}{(c-a)^2}\ge2$
$=>\dfrac{a^2+b^2}{(a-b)^2}+\dfrac{b^2+c^2}{(b-c)^2}+\dfrac{c^2+a^2}{(c-a)^2}\ge\dfrac{5}{2}$ (2)
Từ (1);(2) => đpcm thui
dấu = khi 1 cái =0 2 cái còn lại tổng =0 :pe



#202494 Bất đẳng thức

Đã gửi bởi cvp on 23-06-2009 - 06:39 trong Bất đẳng thức - Cực trị

à quên thực tế thì ko cần cm B>0 ko ảnh hưởng tới bài giải mà! (*)
Vậy bài nè chỉ 3 dòng thui!
Em dùng bổ đề như tn post lên nha!



#202493 Bất đẳng thức

Đã gửi bởi cvp on 23-06-2009 - 06:35 trong Bất đẳng thức - Cực trị

Em nghĩ ra 1 bài này :Cho $a,b,c > 0$ và$ a^{2}-ab+b^{2}\leq c^{2}$.CMR:
$\dfrac{c^{2}}{ab}\geq [\dfrac{2(ab+c^{2})+a(c-a)+b(c-b)}{a^{2}+b^{2}+c(a+b)}]^{2}$

Chứng minh như sau:
Bất đẳng thức cần chứng minh tương đương$\dfrac{{c^2 }}{{ab}} \ge \left[ {\dfrac{{c^2 + c\left( {a + b + c} \right) - \left( {a^2 + b^2 - ab} \right)}}{{ab + c\left( {a + b + c} \right) - \left( {a^2 + b^2 - ab} \right)}}} \right]^2 $
Để ý rằng:$\begin{array}{l}
a^2 + b^2 - ab \le c^2 \Rightarrow \left\{ \begin{array}{l}
ab \le c^2 \\
c^2 + ab \ge a^2 + b^2 \\
\end{array} \right. \\
\Rightarrow B = c\left( {a + b + c} \right) - \left( {a^2 + b^2 - ab} \right) \ge ca + bc > 0 \\
\end{array}$
ta cần cm:$\dfrac{{c^2 }}{{ab}} \ge \left[ {\dfrac{{c^2 + B}}{{ab + B}}} \right]^2 \Leftrightarrow B^2 \left( {c^2 - ab} \right) \ge 0$
Điều nè hiển nhiên đúng!
Dấu = khi a=b=c



#202083 Bất đẳng thức

Đã gửi bởi cvp on 20-06-2009 - 17:35 trong Bất đẳng thức - Cực trị

CHo a,b,c ko âm và $a^{2}+b^{2}+c^{2}=1$.TÌm GTNN :$ P=\dfrac{a}{bc+1}+\dfrac{b}{ac+1}+\dfrac{c}{ab+1}$

Bài nè không khó;để ý rằng
$\begin{array}{l}
a + abc \le a + \dfrac{{a(b^2 + c^2 )}}{2} = 1 - \dfrac{{\left( {a - 1} \right)^2 \left( {a + 2} \right)}}{2} \le 1 \\
\Rightarrow \dfrac{a}{{bc + 1}} = \dfrac{{a^2 }}{{abc + a}} \ge a^2 \\
\end{array}$
tương tự cộng lại là xong!
Bổ sung: bài toán này có thể tìm đc cả max nữa bằng $\sqrt 2 $



#202079 Bất đẳng thức

Đã gửi bởi cvp on 20-06-2009 - 17:22 trong Bất đẳng thức - Cực trị

còn bài này ạ,!

Bài này để anh làm nha:
$\dfrac{{a^3 }}{b} + ab \ge 2a^2 ;\dfrac{{b^3 }}{c} + bc \ge 2b^2 ;\dfrac{{c^3 }}{a} + ac \ge 2c^2$
mà $a^2 + b^2 + c^2 \ge ab + bc + ca$
nên VT≥ab+bc+ca
$ \Rightarrow 2VT \ge \dfrac{{a^3 }}{b} + bc + \dfrac{{b^3 }}{c} + ca + \dfrac{{c^3 }}{a} + ab \ge 2a\sqrt {ac} + 2b\sqrt {ba} + 2c\sqrt {cb} $
=>đpcm Dấu = xãy ra khi a=b=c.


Cố lên em anh thấy em giỏi lắm!
…………Tam Dương…………



#202688 Bất đẳng thức

Đã gửi bởi cvp on 24-06-2009 - 12:29 trong Bất đẳng thức - Cực trị

Bài này, em cũng nghĩ ra , anh Dũng làm đi nhé

ừ anh nghĩ rùi nhưng ko thể cm đc đâu em!ví dụ em chọn a=b=c=0,1 là sai rùi.
em thử xem lại hộ anh nhé!



#202756 Bất đẳng thức

Đã gửi bởi cvp on 24-06-2009 - 20:01 trong Bất đẳng thức - Cực trị

Góp vui bài toán cho topic nè:
Cho $x,y,z$ là các số thực dương.Chứng minh rằng:
$\dfrac{1}{x^2+xy+y^2}+\dfrac{1}{y^2+yz+z^2}+\dfrac{1}{z^2+zx+x^2}\ge\dfrac{9}{(x+y+z)^2}$



#203118 Bất đẳng thức

Đã gửi bởi cvp on 27-06-2009 - 18:23 trong Bất đẳng thức - Cực trị

1,Cho $ a^{3}+b^{3}+c^{3}=3abc+1$ .Tìm GTNN của: $ S=a^{2}+b^{2}+c^{2}$
Xem ai có cách ngắn nhất và sơ cấp nhất cho bài 1
2, Với $a,b,c>0$ sao cho $abc=3$.Tìm GTNN của : $P=\dfrac{\sqrt{a^{2}+b^{3}+c^{4}}}{a^{3}+b^{2}+c}$

Cứ thử bài 1 đã :oto:
Ta có: $1=a^3+b^3+c^3-3abc=(a+b+c)(a^2+b^2+c^2-ab-bc-ca)$
$=>1=(a+b+c)^2(a^2+b^2+c^2-ab-bc-ca)(a^2+b^2+c^2-ab-bc-ca)\le\dfrac{((a+b+c)^2+2(a^2+b^2+c^2-ab-bc-ca))^3}{27}$
$=>1\le(a^2+b^2+c^2)^3$
Do đó: $a^2+b^2+c^2\ge1$
Vậy $(a^2+b^2+c^2)min=1$
p/s:cách nè chắc là sơ cấp nhưng ngắn hay ko thì em ko biết :oto:



#202893 Bất đẳng thức

Đã gửi bởi cvp on 25-06-2009 - 20:10 trong Bất đẳng thức - Cực trị

nản,đòi hỏi cao quá đấy,xem ở đây đi:
http://www.mathlinks...ic.php?t=281905

đc rùi em biết thêm mấy cách thanks pác



#202882 Bất đẳng thức

Đã gửi bởi cvp on 25-06-2009 - 19:51 trong Bất đẳng thức - Cực trị

$VT \ge \left( {\dfrac{{4(xy + yz + zx)}}{{{{(x + y + z)}^2}}}} \right)\left( {{{\left( {\dfrac{1}{{x + y}}} \right)}^2} + {{\left( {\dfrac{1}{{y + z}}} \right)}^2} + {{\left( {\dfrac{1}{{z + x}}} \right)}^2}} \right) \ge \dfrac{9}{{{{(x + y + z)}^2}}}$

pác nè làm vắn tắt wa.Hic mà cm bài toán nè lại phải dùng một bài toán khác cũng mạnh và khó như bđt Iran 96 sao pác!em nghĩ dùng cách khác đi



#202856 Bất đẳng thức

Đã gửi bởi cvp on 25-06-2009 - 17:22 trong Bất đẳng thức - Cực trị

anh duca1pc giải bài này bằng đại số đi ạ :D,mà anh giải cách lượng giác ra luôn đi,em hok quen lượng giác lắm

Mình ghi lời giải lượng giác cho bạn nè.
Từ giả thiết suy ra tồn tai tam giác ABC nhọn sao cho: $a=2cosA;b=2cosB;c=2cosC$
(Vì $cos^2A+cos^2B+cos^2C+2cosAcosBcosC=1$ mà)
Vậy $a+b+c\le 3 <=>cosA+cosB+cosC\le \dfrac{3}{2}$ cái nè là bđt cơ bản của lượng giác
Cái bđt thứ 2 thì nhớ lại bổ đề sau $xycosA+yzcosB+zxcosC\le \dfrac{(x^2+y^2+z^2)}{2}$
Ta có: $\dfrac{ab}{c}+\dfrac{bc}{a}+\dfrac{ca}{b}\ge a^2+b^2+c^2$
<=> $\dfrac{cosAcosB}{cosC}+\dfrac{cosBcosC}{cosA}+\dfrac{cosCcosA}{cosB}\ge 2(cos^2A+cos^2B+cos^2C)$
Đến đây có thể thấy nếu đặt $ \sqrt{\dfrac{cosAcosB}{cosC}}=y; \sqrt{\dfrac{cosBcosC}{cosA}}=z; \sqrt{\dfrac{cosCcosA}{cosB}}=x$ thì bất đẳng thức trên quy về bổ đề vừa nêu!
=> đpcm!



#203128 Bất đẳng thức

Đã gửi bởi cvp on 27-06-2009 - 19:53 trong Bất đẳng thức - Cực trị

Anh Dũng có cách khá ngắn gọn và hay đó. Đây là cách của em:Đặt $ab+bc+ac=x.$
$1=(a^3+b^3+c^3-3abc)^{2}=[(a+b+c)(a^2+b^2+c^2-ab-bc-ca)]^{2}=(S+2x)(S-x)^{2}=S^{3}+2x^{3}-3Sx^{2}$
$ \Leftrightarrow S^{3}=1+x^{2}(3S-2x)\geq 1$ vì $3S>2S=2(a^{2}+b^{2}+c^{2})\geq 2(ab+bc+ac)=2x$.Vậy $Min S=1$

ừ cách em cũng hay đó :pe